LSAT and Law School Admissions Forum

Get expert LSAT preparation and law school admissions advice from PowerScore Test Preparation.

 Administrator
PowerScore Staff
  • PowerScore Staff
  • Posts: 8916
  • Joined: Feb 02, 2011
|
#71265
Please post your questions below! Thank you!
 medialaw111516
  • Posts: 80
  • Joined: Dec 11, 2018
|
#72162
Why is C wrong? Isn't the store owner saying that because of what he heard from shoppers at his store he believes they are not worried about crime in the neighborhood? Wouldn't that be the generalizing in C?
 endlessly160
  • Posts: 2
  • Joined: Nov 22, 2019
|
#72212
I would appreciate some feedback regarding the structure of the argument presented here. As I understand it:

Premise: everyday I talk to people who shop at my store and they tell me they are not worried [about crime in this neighborhood]

Intermediary conclusion: my customers are not worried about crime in this neighborhood

Conclusion: crime is not adversely affecting my business by reducing the number of people willing to shop at my store

The conclusion seems to come out of nowhere because it talks about crime reducing the number of customers, which confused me for quite a while regarding how to pick apart this argument. Eventually I arrived at something quite broad, like: how do you know your number of customers aren't decreasing just from talking to people who come in?
In the moment I couldn't put my finger on exactly how the sample was biased, so I decided against the correct answer choice D, and went to B thinking that the owner was simply connecting things that weren't related, hence appealing to personal opinion. Could someone please clarify the way in which the owner's sample is biased, and what's wrong with choice B? Thank you.
 Jeremy Press
PowerScore Staff
  • PowerScore Staff
  • Posts: 1000
  • Joined: Jun 12, 2017
|
#72260
Hi medialaw,

Answer choice C is wrong because the conclusion is not about the effect of crime on the "whole neighborhood." It's only about the effect of crime on the store owner's store. In order for answer choice C to be correct, the store owner would have to conclude (based on what's happening at his or her store) that crime must not be adversely affecting the neighborhood as a whole.

Hi Endlessly160,

You've correctly analyzed the structure of the argument. And your prephrase was right on: it's about the numbers, specifically the fact that the store owner isn't talking to enough people! That's a "sampling" issue. The "sample" of potential shoppers the store owner obtained is biased toward especially brave shoppers who are willing to come out in spite of any crime. The sample is necessarily missing any shoppers who aren't brave enough to come to the store.

The reason answer choice B is not a flaw here is that the personal opinions of the shoppers (about whether crime in the neighborhood worries them) are actually relevant to the portion of the conclusion that identifies people "willing to shop at the store," which is a factual claim that depends on those people's opinions about (or reactions to) the level of crime in the neighborhood. So it's not that personal opinions are irrelevant. It's just that the store owner hasn't obtained a sufficient sampling of those opinions.

I hope this helps!

Jeremy
 LSAT-Learner
  • Posts: 4
  • Joined: May 15, 2020
|
#75833
Can you explain why E is incorrect? Thank you
 Luke Haqq
PowerScore Staff
  • PowerScore Staff
  • Posts: 747
  • Joined: Apr 26, 2012
|
#75881
Hi LSAT-Learner!

Happy to address why answer choice (E) is incorrect. Since this is a flaw in the reasoning type of question, it's important to break the stimulus down into premises and conclusions to see what the author's argument is, and where the holes in it are. The author makes the following conclusion (the word "so" often suggests a conclusion, much like "thus" or "therefore"):
Conclusion: "So crime is not adversely affecting my business by reducing the number of people willing to shop at my store."
How does the author arrive at this conclusion? The author uses two premises in support of this conclusion, the first explicitly stated and the second an implicit assumption:
P1: "Every day I talk to people who shop at my store, and they tell me that they are not worried."
(P2): Customers not worried :arrow: No reduction in total number willing to shop at store
In other words, the business owner is saying that crime hasn't caused a reduction in numbers at the store and backs this up by noting that those shopping at the store aren't afraid (if they're not afraid, then no reduction in numbers). While the store's current customers might not be dissuaded by crime, however, this line of reasoning fails to take into consideration the possibility that other people who are not yet customers might be dissuaded and not shop at the store. If this were true, then the business owner's conclusion does not follow--crime might still be adversely affecting the business by reducing the number of people willing to shop there.

Answer (E) states that the flaw is that the argument "fails to consider that crime might affect the neighborhood negatively without affecting businesses negatively." This answer is problematic because it does not get to the author's conclusion, which is about numbers (numbers of people who shop at the store). The author doesn't appear to assume one way or another as to whether crime might affect neighborhoods negatively without affecting businesses negatively--this could be true (or false), but either way, one would still be left wondering as to the particular steps that the author/business owner makes in this argument, namely, of assuming that no reduction in existing customers means no reduction in overall numbers. Such argumentation, again, fails to consider that crime might be dissuading new visitors from frequenting the store.

Get the most out of your LSAT Prep Plus subscription.

Analyze and track your performance with our Testing and Analytics Package.